Download as pdf or txt
Download as pdf or txt
You are on page 1of 55

Annotation Summary for Nbme 24 Explanations:

Page 1:
FreeText
D. Page 570 (636) of FA2019.

GFR = Clearance of inulin.


Normal GFR = 100 mL/min

Creatinine clearance overestimates GFR because creatinine is excreted by tubules.

Remember the definitions. The afferent arteriole brings many nutrients and substances to
the glomerulus, where filtration happens based on pressures.
Then along the tubule, some things are reabsorbed (all sugar, proteins, etc in PCT), while
some things are secreted (PAH).

Everything that was filtered and secreted gets excreted (clearance). Inulin is neither
reabsorbed nor secreted so it is freely filtered and best indicates GFR.
Creatinine is secreted by the kidney, so it overestimates GFR.

Page 2:
FreeText
A. Page 159 (164) FA2019
Onchocerca volvulus by the female black fly.
The other answer chocies are generally related to bacteria or unrelated to any nematodes.
This child is presenting with practically no symptoms besides the four black skin nodules
indicative of O volvulus.

Page 3:
FreeText
E. Page 467 (525) FA2019.
Nevus simplex. Not in First Aid.

This is not a Strawberry Hemangioma, which would be raised or nodular. But if you did
think it is a strawberry hemangioma, then that also regresses.
This patient has a nevus simplex. Another similar lesion is a stork bite (port-wine stain).

These lesions disappear/regress by age 5-6 years.

Page 4:
FreeText
E. Page 83 FA2019.

This question is a simple biochemistry question regarding mechanisms of managing


hypoglycemia.
Cortisol is one of the defenses against hypoglycemia.
The question asks for an enzyme in the adrenal medulla, which synthesizes NE and Epi.
Choice E is the only relevant enzyme. It works with SAM to convert NE to Epi and is
upregulated by cortisol.

You should know exactly what the other enzymes are for.
Acetyl-CoA carboxylase is the first enzyme for fatty acid synthesis and takes Acetyl CoA
and yields Malonyl CoA.
Choice B is related to homocysteine becoming methionine and is related to homocystinuria.
Choice C is a relevant enzyme because it makes the SAM that PMNT uses it is not
upregulated by cortisol.
Choice D, a "racemase" is probably an enzyme youve never heard of. It is an epimerase
involved in fatty acid and amino acid breakdown.
Stamp
Image

Page 5:
FreeText
B

Very simple and straightforward.


Patient is unresponsive, has no EEG activity, and no purposeful movement.
The question states "consistent with its directives" so her husband is simply requesting
what the patient wanted.

The patient does not have a DNI or DNR since she was intubated. She must just be
allowed to die rather than sustaining her body alive.

Page 6:
FreeText
B. Page 510 (570) and page 491 (571).

This patient has a communicating hydrocephalus, as is evident by the fact that there is no
obstruction that is impeding the CSF flow.
The patient has a widening of the subarachnoid space because the arachnoid granulations
are not absorbing the CSF, most likely due to inflammatory injury.
Blood is inflammatory. Her leaking aneurysm has caused inflammation and damaged the
arachnoid granulations.
The arachnoid granulations absorb CSF and send it to the dural venous sinuses, which
empty into the internal jugular vein.

The other choices also do not make much sense.

The choroid plexus does not absorb CSF, it produces CSF.


Choice C is indicative of a noncommunicating (obstructive) hydrocephalus.
Stamp
Image

Page 7:
FreeText
E. Page 580 (649) FA2019.
This patient has a non-anion gap metabolic acidosis that she is immediately compensating
for with hyperventilation.
You can remove A, C, D, and F since those are part of the MUDPILES.
Choice B would present with malabsorption symptoms.
IBD patients tend to have reduced Na and Cl absorption and increased K secretion.

Page 8:
FreeText
B. Page 571 (638) and page 295 (317) FA2019
Glomerular dynamics and autoregulation are high-yield.
For the kidney there are myogenic and tubuloglomerular feedback mechanisms of
autoregulation.
The myogenic mechanism is where the afferent arteriole controls blood flow purely based
off of blood pressure entering the kidney.
The tubuloglomerular feedback mechanism involves the JGA and a paracrine signaling
mechanism utilizing ATP, adenosine, and NO.

For our patient, there is decreased blood entering the renal artery.
They also mention that the autoregulatory mechanism will preserve GFR and RPF.
If you constrict either arteriole, you will alter the GFR and the RPF, so delete choices D and
E.

Afferent arteriole dilation (usually caused by prostaglandins, PDA) usually increases GFR
and RPF but since perfusion into the kidney
is reduced, nothing will be severely altered.

Page 9:
FreeText
B. Page 156 (161).
Toxoplasma gondii.
The mother is fine. The baby has the triad of chorioretinitis, hydrocephalus (head
circumference is in 3rd percentile), and intracranial calcifications.
In the USA, ingestion of undercooked pork is the most likely answer choice.

Oocysts in cat feces are dangerous so pregnant women should avoid cat litter.
But even if contact with cat litter is an answer choice, cysts in meat is the most common
route of infection and more likely.

Page 10:
FreeText
C. Page 289 (308) FA2019).
The midsystolic murmur that radiates to the carotid arteries bilaterally is indicative of aortic
stenosis.
The SAD symptoms of syncope, angina, and dyspnea are a result of not being able to
increase CO when there is demand.

Page 11:
FreeText
E. Page 360 (401) FA2019.
This patient has internal hemorrhoids, which are above the pectinate line. That area has
the superior rectal vein (suprior above).
Below the pectinate line are external painful hemorrhoids, which have the inferior and
middle rectal veins (inferior below). Easy.

Page 12:
FreeText
C. Page 257 (268) FA2019.
Very straightforward question with an ideal table.
Plug and chug.

Specifcity is the TNrate, which is 1- FPrate, which is TN/(TN + FP).


Specifcity is the TNrate, which is 1- FPrate, which is TN/(TN + FP).
118/ (118+6) = 118/124 = 95%

Page 13:
FreeText
B. Page 262 (274) FA2019.
The "likelihood of missing an association" is a false-negative. Stating there is no effect or
difference when there reall yis one is beta.
Type 2 error is the beta value itself, 0.10 which is 10%. Very straightforward question.

Do not confuse this with power, which is 1-beta, and that would be 90% here.

Page 14:
FreeText
A. Page 475 (533) FA2019.
Bisphosphonates have significant side effects such as esophagitis, osteonecrosis of the
jaw, and atypical femoral stress fractures.
It is important to take this medication with abundant water and to remain upright after taking
it.

Page 15:
FreeText
E. Page 305 ( 331) FA2019.
Pregnancy is one of the risk factors for dilated cardiomyopathy.
There is no problem with her valves or blood pressure (A and B are the same answer).
An increase in pulse or HR would not cause a murmur. And she does not have pulmonary
HTN (choice C).

Dilated cardiomyopathy increases stroke volume.

Page 16:
FreeText
B. Page 612 (689) FA2019. Also -- https://www.ncbi.nlm.nih.gov/pmc/articles/PMC4896089/
The physiology of an erection involves cerebral control, autonomic control, and molecular
mechanisms. Cerebrally controlled erections are induced by erotic visual stimuli or
thoughts,
which is related to choices A, C, and E. The medial preoptic area (memorized as medial
penis area
and the penis is medial) and paraventricular nucleus of the hypothalamus are the main
structures
involved. Dopamine is the most important brain neurotransmitter for an erection, most likely
through its stimulation of oxytocin release.

Autonomic control is through parasympathetic stimulation and central suppression of the


sympathetic nervous system. The sacral S2-S4 provide parasympathetic supply through
the
sacral reflex arc that mediates an erection through tactile penile stimulation, which would
work
in our patient.

Patients with a lesion above T9 cannot maintain a psychogenic erection because central
suppression to the sympathetic stimulation is necessary for that. Patients with sacral spinal
cord
injury may maintain a psychogenic erection but it will be weaker than normal.

The molecular control is through NO such as through the use of viagra.


Stamp
Image

Page 17:
FreeText
C. Page 382 (425) of FA2019.
First of all, they meant MSH2 (typo).
This question is about Lynch syndrome, which usually has extensive family history.
The 3-2-1 rule of 3 relatives in 2 generations with 1 being under 50 (such as the patient
himself) will all have
history of CRC. The most important thing here is that this is NON-POLYPOSIS so there will
be no polyps.

The gross image is a beautiful example of what would appear as an "apple core" lesion on
barium enema x-ray.
HNPCC is related to endometrial, ovarian, and skin cancers and there are many genes
involved such as
MSH2, MLH1, MSH6, and PMS2.

Page 18:
FreeText
C. Page 516 (576) FA2019.
This patient is presenting with Parinaud syndrome, which has a vertical gaze palsy due to
compression of the tectum.
She most likely has a pinealoma. This is a very straightforward and simple question.

In males, this can lead to precocious puberty and they would have elevated hCG because
this is similar to germ cell tumors like a seminoma.

Page 19:
FreeText
A. Page 130 FA2019.
Conjugation is when two bacteria mate through a conjugal bridge and transfer a plasmid
from one to the other.
Here the E coli is resistant to ampicillin and the Salmonella is resistant to streptomycin.
They are cultured together.
Now the Salmonella is resistant to both antibiotics. This is because the E coli gave its
ampicillin resistance plasmid to the Salmonella.

Gene duplication and point mutation are irrelevant.

Transduction involves a virus.

Transposition is an event that occurs within the same one bacteria, not between 2 different
bacteria.

Page 20:
FreeText
E. Page 117 FA2019.
This patient has LAD1, a deficiency in LFA1 integrin CD18. Autosomal recessive.
Patients present with recurrent skin and mucosal bacterial infections due to phagocyte
dysfunction.
There is impaired migration and chemotaxis. Delayed separation of the umbilical cord is
usually a notable problem.

Page 21:
FreeText
D. Page 632 (711) of FA2019.
She has a SL tumor that resembles testicular histology and produces androgens, leading to
virilization.
The patient's presentation is classic for this tumor.

Choices A (cortisol) and B (aldosterone) are irrelevant and this patient does not have a
CAH.
Choices E (prolactin and GH) and F (FLAT) are irrelevant and this patient does not have a
pituitary adenoma.

Page 22:
FreeText
D.
Pretty obvious. The pain was of sudden-onset. It started in the left flank and moved to the
scrotal area because the stone travelled to the penis.
No palpable masses in the abdomen or scrotum rules out any tumors. No rebound
tenderness rules out anything related to the appendix.

Classic "loin to groin" pain of nephrolithiasis.

Page 23:
FreeText
D. Page 588 (660) FA2019.
This patient has stress incontinence, as is evident from her urine leakage after extensive
intra-abdominal pressure from a cough or sneeze.
The problem is due to urethral hypermobility or the intrinsic urethral sphincter.
Kegel exercises strengthen the pelvic floor. The main targets for treatment are the levator
ani muscles; iliococcygeus, pucococcygeus, & puborectalis.
Pessaries are another treatment option.

Regardless, any exercise will generally strengthen skeletal muscle. The internal anal
sphincter is smooth muscle under autonomic control.
It will not strengthen from exercise.
The external anal sphincter, on the other hand, is under pudendal nerve control (feces
leakage during delivery) and would strengthen from exercise.

Page 24:
FreeText
C. Page 82 and 85 FA2019. Page 85 of FA2020.
This patient has hyperammonemia. Everything else is alright though.
This is most likely some organic acid metabolism issue. There are plenty, like propionic
acidemia, methylmalonic acidemia, etc.

In methylmalonic acidemia patients have hypoglycemia (accumulation of methylmalonate


In methylmalonic acidemia patients have hypoglycemia (accumulation of methylmalonate
inhibits gluconeogenesis), ketosis (to make up for
hypoglycemia), and hyperammonemia (accumulation of methylmalonate inhibits the urea
cycle).

Page 25:
FreeText
E. Page 159 (164) FA2019.
This patient has the classic symptoms of Trichinella spiralis.
The nematode travels the bloodstream and enters striated muscle.

Page 26:
FreeText
B. Page 432 (478).
Methotrexate is a folic acid analog that competes with folate and prevents
DHF reductase from making dTMP, inhibiting DNA synthesis to treat cancers.

If you give these patients concomitant folic acid, it will not work since
methotrexate will just compete with it.
Leucovorin is 5-formyl-THF that works downstream from DHF reductase,
bypassing the step that methotrexate inhibits.
Leucovorin effectively works like folate and prevents the myelosuppression.

In comparison, you must give leucovorin the patients taking 5FU because
leucovorin increases the binding of 5FU with its target, thymidylate synthase.
Stamp
Image

Page 27:
FreeText
D.
The patient is clearly having difficulty with visual association. She is seeing things but not
sure what it is related to until touching it.
This is prosopagnosia due to a PCA stroke. Since its a visual problem for the most part, but
neurologic exam is fine otherwise, you can choose PCA.

ACA has loss of sensation and paralysis in legs, while MCA has loss of sensation and
paralysis in the face and arms.
Lenticulostriate is related to the striatum and internal capsule and often has lacunar infarcts
due to hypertension. Would just have paralysis.
Anterior choroidal would have loss of sensation and paralysis also.

Page 28:
FreeText
B. Page 330/360 discusses PTH. Page 341/373 discusses PTH disease.
This patient's parathyroid glands were probably removed along with the thyroidectomy.
PTH is phosphate trashing hormone, so without PTH the phosphate will be high. PTH is
obviously gonna be low.
Vitamin D is not related to PTH levels.

Page 29:
FreeText
D. Page 508/568.
This patient has the familial form of Alzheimer's disease (neurofibrillary tangles of tau
protein, neuritic plaques of beta amyloid,
and neuronal loss due to cortical atrophy). The familial form is due to presenilin1 and 2 as
well as APP.

Page 30:
FreeText
D. Page 518/601.
Area B is the dorsal root ganglion.
This area carries afferents.
This is basically what you check for with the knee jerk.
Hence, that reflex would be missing in severe DRG damage.
Stamp
Image

Page 31:
FreeText
C. Page 417/461.
Very straightforward question about heme synthesis.
The first step involving ALA synthase makes ALA by combining glycine and succinyl-CoA.

Page 32:
FreeText
C. Page 258/269.
The odds ratio tells you the odds of something being related to another thing. Used in case-
control studies. (Case-contORl).
When the odds ratio is 1, then both things are equally likely. That means, that eating a
cookie, or drinking milk are equally likely to cause E coli.
When the odds ratio is more than 1, then the relevant exposure is related to the outcome.
Drinking milk continuously has an odds ratio of 1.0 but the top right (ate cookies, did not
drink milk) has an OR=6 so choose C.
FreeText
MISTAKE. OR= 6.0

Page 33:
FreeText
C. Page 357/397 FA2019.

The mesenteric circulation consists of three primary vessels that supply blood to the small
and large bowel: the celiac artery,
superior mesenteric artery (SMA), and inferior mesenteric artery (IMA).
Blood flow through these arteries increases within an hour after eating due to an increase
in metabolic demand of the intestinal mucosa.
Chronic occlusion of a single vessel allows collateral blood flow to compensate, thus
symptoms do not typically present until at least two
primary vessels are occluded.

The 3-cm ectatic aorta means she has an abdominal aortic aneurysm.
Generally, they tend to occur below the renal branches so the two closest branches are
SMA (slightly above renal) and IMA (below renal).
Also, splenic flexure is a common watershed area supplied by SMA and IMA so
involvement of both can cause her symptoms.

Page 34:
FreeText
C. NOT in FA.
NBME likes asking about kids abusing glue inhalation.
Any child with an unexplainable suspicious rash on the face and a decline in intelligence is
most likely inhaling glue.
Below are people who were caught huffing inhalants such as glue, paint, markers, and
other solvents.
Stamp
Image
Stamp
Image
Stamp
Image

Page 35:
FreeText
C. Pg 309/337.
The patient is presenting with pulsus paradoxus, which is a drop in systolic BP by
>10mmHg during inspiration.
Recall that during inspiration, blood returns to the right heart.
In cardiac tamponade, fluid around the heart prevents expansion of the heart so it is unable
to sustain the increased return.
This is why the patient has JVD as well.
Stamp
Image
FreeText
Tamponade is french for compression. All the chambers are being compressed
by fluid in the pericardial space (red arrows in image at bottom). As this continues
the cardiac chambers decrease in size and diastolic filling is impaired, one part of
Beck triad; hypotension. Back up of blood causes distended neck veins.
Distant heart sounds are due to muffling of sounds by fluid.

Usually when we take a breath in, increase in venous return to the right heart
increases right heart volume. Usually the increased volume of RA distends into
pericardial space, but in tamponade that is not possible so the RV pushes the
IV septum into the LV, decreasing volume in the LV and that means you have
significant drop in systolic BP of more than 10mmHg because the LV will have
trouble pumping into the aorta. You also note JVD. In asthma and other
obstructive lung diseases, there is amplification of intrathoracic pressure
variation during respiration, which is then transmitted to the distal arteries.

Page 36:
FreeText
E. Page 146/148.
V cholerae other V vulnificus as well as V parahaemolyticus are all related to consumption
of undercooked seafood like shellfish or oysters.

B cereus is related to reheated rice and usually causes vomiting but can also cause
diarrhea within 8-18 hours.
C jejuni is related to undercooked meat or contact with animals. Plus this causes bloody
diarrhea.
C perfringens is related to food left standing, allowing the spores to germinate and create
heat-labile enterotoxin. Gas gangrene is a complication.
S aureus is related to mayonnaise and dairy products and would have rapid onset diarrhea.

Page 37:
FreeText
B. Page 114 FA2019.
This patient clearly had transfusion related acute lung injury (TRALI).
TRALI occurs due to a 2 hit mechanism with neutrophil priming and then neutrophil
activation leading to pulmonary edema.
The patient is otherwise fine.
This topic was in NBME 21 and NBME 22 as well.

Page 38:
FreeText
B. Page 471/529.
This question about desmosomes is relevant to pemphigus vulgaris, which has antibdoies
against desmogleins that make desmosomes.
Loss of desmosomes separates the keratinocytes from the stratum spinosum.
Stamp
Image

Page 39:
FreeText
A. Page 310/338.
This patient most likely has Behcet syndrome. The image is perfect. So is the question
stem.
Herpangina is due to coxsackie A and causes oral ulcers on the palates.
Lack of genital or skin lesions rules out HSV but that is not even an answer choice, how
nice of them.
Herpes zoster would have a rash following a half-body dermatome in some
immunocompromised elderly person.
Secondary syphilis causes gummas, irrelevant here.

This patient simply has recurrent aphthous ulcers that come and go and could possibly be
due to HSV or parvovirus.
Stamp
Image
Stamp
Image
FreeText
Herpangina

Page 40:
FreeText
E. Page 664
Even with incomplete understanding of what is happening, you come down to choices B
and E.
The gram negative rods causing her UTI most likely are E coli. She might have E coli
sepsis leading to ARDS.
The ARDS is due to pulmonary capillary leakage. Noncardiogenic pulmonary edema.
The patient clearly has a low pH (acidosis) and the low pCO2 is due to hyperventilation as
a result of the hypoxemia.

Page 41:
FreeText
B. Ethics.
Give away question.
More discussion is needed as to why there are continuous fights during meals and if the
parents might be eventually divorcing.
Is the issue a lack of understanding how to manage the childs disease? Or are the parents
falling apart?

Page 42:
FreeText
E. Page 364/405 FA2019.
This patient definitely does not have A, B, or C.
Fecal impaction is the only other plausible thought amidst the exhaustion of an exam and
amidst this terrible NBME board of question writers.
However, fecal impaction would be in the colon or rectum, not the groin as in this patient.
The groin is where inguinal hernias occur.
This is an extremely old patient so this might be a direct inguinal hernia.
However, given the fact that this was not reducible and it is incarcerated, this might be a
femoral hernia.

Page 43:
FreeText
D. Page 315/344 FA2019.
This is a straightforward question about the MOA of statins.
Statins inhibit the conversion of HMG CoA to mevalonate, a cholesterol precursor.
Now you start running out of that and cant make cholesterols in the liver, so you make
more LDL receptors to pick up cholesterols into the liver.
You also ramp up HMGCoA reductase production. This medication can be hepatotoxic.

Page 44:
FreeText
D. Page 518/578.
This question is simply a matter of eliminating the other choices.
ALS would have no sensory or bowel/bladder deficits, so this was an excellent bait.
However, ALS is not about weakness of muscles.
ALSO has UMN and LMN degeneration that causes a large array of symptoms, not just
neck and extremity muscle weakness.
Parkinson is irrelevant in this question. Patient would have tremor or rigidity.
Poliomyelitis is irrelevant (pg 19/579). This would present with asymmetric LMN weakness
and respiratory failure.
Syringomyelia is irrelevant. Patient would have symmetrical loss of pain and temperature in
the cape distribution.

Page 45:
FreeText
C. Page 304/330.
Not the worst question but definitely intimidating due to lack of experience with such a
case.
The patient had RCA stenosis that was dealt with and she was fine for 3 months.
Something must have been going on in those 3 months because then she started a 2
month period of angina with exertion.
Its like her RCA stenosis came back again! And only choice C fits that explanation.
Choices A and B fail because she is fine when at rest. Same for choices D and E.
Restenosis occurs from gradual narrowing of the stent lumen due to neointimal
proliferation.
This is why patients are put on a regiment of antiplatelet therapy after stent placement.

Page 46:
FreeText
Page 114 FA2019.
Quesiton 37 was the one about transfusion related acute lung injury.
This patient here has the acute hemolytic transfusion reaction due to ABO incompatibility.
This is a type 2 HSR (antibodies and antigens) not a type 1 (allergic/anaphylactic).

Page 47:
FreeText
F. Page 689/767 FA2019.
This patient never smoked and is a female, so that fits the profile for adenocarcinoma.
The incidental finding of her coin lesion (rather than her presenting because of unexplained
weight loss and cough) fits the profile for a hamartoma.

The microscopic examination is diagnostic. The rosettes are almost always indicative of a
neuroendocrine turmor.
They are most likely referring to a bronchial carcinoid tumor. The question did not want a
precise diagnosis anyway so its not 100% clear.

Page 48:
FreeText
A. Page 67.
This is clearly an extreme presentation of the "Casal necklace" rash that occurs in the C3/
C4 dermatome.
This patient has vitamin B3 deficiency. Niacin is a part of NAD. The A is adenine.

Page 49:
Stamp
Image
Stamp
Image
FreeText
A. Page 610/686.
This artery was previously tested in some NBME as the
deferential artery.
You can answer this question by simply thinking of what
other artery would be able to supply the testis.
The other arteries labelled do not reach the testis.

Page 50:
FreeText
C. Page 82 and 83 FA2019.
This is a straightforward question on OTC deficiency.
High orotate and hyperammonemia with a low BUN.
The excess carbamoyl phosphte upregulates
pyrimidine synthesis, forming excess orotate.

Page 51:
FreeText
E. Page 346/379.
This patient has diabetic ketoacidosis, which is most commonly seen in DM1 patients that
missed their insulin.
The body starts breaking down fats to increase ketogenesis, which is great for them.
However, the extremely high glucose levels cause severe dehydration that can lead to
delirium.
The osmotic diuresis needs to be managed with IV fluids along with the IV insulin and K+.

Page 52:
FreeText
A. Page 256/266.
Classic NBME trash. The question asks for a time-efficient, experimental design.
While case-control studies are time-efficient, they are observational studies.
You would need a case (people drinking high arsenic) and control (people with no arsenic).

Experimental studies are usually randomized controlled trials, where E would be the best
answer.

Page 53:
FreeText
F. Page 335/366.
This patient must have a pheochromocytoma, which is treated with an irreversible alhpa
blocker (to stop vasoconstriction), followed by a
beta blocker (to decrease HR but these also cause vasoconstriction so they are given a
week after the alpha blocker is initiated).

Page 54: concentrations of anti-endomysial and anti-tissue transglutaminase


FreeText antibodies are increased patient's serum antibodies are diagnostic of Celiac
E. Page 375/416. disease.
This patient's serum antibodies are diagnostic of Celiac disease.
The answer choice was very easy.

A -- Probably some parasitic infection.


B -- Erosions are irrelevant. Small intestine erosion is related to IBD.
C -- Irrelevant.
D -- Whipple disease.

Page 55:
FreeText
E. Page 315/344.
Statins work by indirectly causing an increase in hepatic LDL receptors.
Same question as question 43 block 1.

Page 56:
FreeText
C. 153/155.
This was the best answer to pick given the image and patient's AIDS history.
This image shows budding but it is not broad-based, its narrow budding.
Crypto is stained with india ink or mucicarcmine and can also be found with latex
agglutination.
This is a heavily encapsulated yeast that causes meningitis, encephalitis, and
cryptococcosis.
Patients inhale this so its found in the bronchoalveolar lavage.

Page 57:
FreeText
C. Page 121 and page 424/469.
This patient's leukocyte count and neutrophil count are very low.
She simply needs something that will help her make new WBCs.
AML is treated with vitamin A and arsenic (if its the APL variant with t(15;17).
The problem in AML is the high amounts of myeloblasts not becoming mature, so vitamin A
and arsenic induce differentiation.

Page 58:
FreeText
F. Page 306/332.
This patient clearly has left heart failure. The fact that the x ray shows cardiomegaly and
auscultation reveals crackles, it is decompensated.
He has orthopnea, paroxysmal nocturnal dyspnea, and the pulmonary edema.
Since the heart is not working well, obviously stroke volume and cardiac output will be low.
The blood remains in theheart so his EDV will be high.

Page 59:
B. Pg 636/716.
NBME has asked about this before too.
I explained that one using this same image.
The anatomical basis of the spread of breast cancer is

FreeText high-yield.
First, recall that bone mets can be lytic or blastic (2
breasts, 2 lesion types).

B. Pg 636/716. The intercostal veins drain into the azygos, which drains
some into the SVC
but a lot of it goes to Batson's vertebral venous plexus as
NBME has asked about this before too. well, causing the mets.
C - Goes to brachiocephacli v.
D- Goes to A, which goes to E, ultimately coming to

I explained that one using this same image. brachiocephalic also.

Regardless, axillary lymph mets is the most important


prognostic factor.
The anatomical basis of the spread of breast cancer is high-yield. This is by the lateral mammary-> lat thoracic -> axillary
veins.

First, recall that bone mets can be lytic or blastic (2 breasts, 2 lesion types).

The intercostal veins drain into the azygos, which drains some into the SVC
but a lot of it goes to Batson's vertebral venous plexus as well, causing the mets.
C - Goes to brachiocephacli v.
D- Goes to A, which goes to E, ultimately coming to brachiocephalic also.

Regardless, axillary lymph mets is the most important prognostic factor.


This is by the lateral mammary-> lat thoracic -> axillary veins.
Stamp
Image
Stamp
Image
Stamp
Image

Page 60:
FreeText
E. Page 590/663.
The patient has CKD.
Without kidney function, you can assume the hormones that work through the kidney will
not be able to carry out their tasks.
For this question, EPO will not work, so hematocrit will be low.
PTH will not work so your body will make more PTH in attempts to get a result. However,
PTH function will fail due to the CKD.
The renal 1 alpha hydroxylase will not function, so 1,25 dihydroxy vitamin D (calcitriol) will
be low.

Page 61:
FreeText
E. Page 41 and page 323/352.
This is just a question about POMC, a prohormone peptide chain, yields the opio beta-
endorphin, melano MSH, and cortin ACTH.
This is all a result of post-translational modification.

This is not an example of alternative splicing, where one gene can yield different mRNA
based on how that mRNA's exons are reorganized.
This is not due to post-transcriptional modification, which is practically what alternative
splicing is.

Page 62:
FreeText
E. Page 332/362.
Histamine is a vasodilator that causes vasodilation through the H1 receptor via IP3 and Gq.
It is released from basophils and mast cells and is a major part of inflammation.
The vasodilation and increased vascular permeability are key characteristics that all
choices A-D fail.

Page 63:
FreeText
E. Page 672/772.
It is straight from the book.
No reason to pick A, beta 2 might cause vasodilation but not bradycardia.
Diaphoresis and dry mouth are muscarinic effects.

Think about the fact that beta blockers help treat essential tremor and can mask the tremor
of hypoglycemia.
So a beta agonist must be capable of causing tremors.

Page 64:
FreeText
D. Page 582/653 and page 591/664.
This 4yo kid most likely had a Strep infection and now has PSGN.
The only GN answer choice is D.
Choices A and B are nephrotic so they are wrong.
Choice E is basically acute interstitial nephritis and choice C is a papillary problem (not
glomerular).
Those two choices are related to various medications or conditions that are irrelevant here.

Page 65:
FreeText
D. Page 647/728.
This baby was born early (28 weeks) and has developed NRDS.
After being given therapeutic 100% O2, the RIB problems can come up.
Straightforward question about the B, bronchopulmonary dysplasia.

Bronchial asthma is irrelevant.


Bronchiectasis and chronic bronchitis are COPD related.
Bronchiolitis obliterans is the result of chronic lung transplant rejection.

Choice F is random. Its a malformation where a lobe forms a cyst that does not function as
normal lung tissue.
https://en.wikipedia.org/wiki/Congenital_pulmonary_airway_malformation
Stamp
Image
Stamp
Image
FreeText
Both images show cystic
changes in the lung.

Page 66:
FreeText
D. Page 59 FA2019.
Men pass their Y chromosome to sons, so there definitely is not X-linked inheritance here
since all men are having it.

Page 67:
FreeText
B. Page 506/566.
This description of pain around one eye is classic for a cluster headache.
The lacrimation this patient has is also key in the diagnosis.

Page 68:
FreeText
A. Page 354/388.
The only way to get bilious vomiting is if there is an obstruction past the 2nd part of the
duodenum.
Careful. Annular pancreas does not always present with bilious vomiting, so it is not a
critical characteristic.

Page 69:
FreeText
B. Page 574/642.
The PCT reabsorbs most things including phosphate.
This child probably has Fanconi.
Very important to spend some time and master these.
Of the various renal tubular defects, only fanconi
causes metabolic acidosis.

This was not a question about the RTAs on page 581/651.


Those cause acidosis of the urine but not metabolic acidosis.
Page 70:
FreeText
B. Page 226/236.
If it is ever the case that smoking might cause the problem, then it is most likely right.
The patient most likely has transitional cell carcinoma (urothelial carcinoma).

Schistosoma haematobium can lead to squamous cell carcinoma of the bladder and
would also present with painless hematuria. But there would be fever.
The image shows transitional cell carcinoma.

Page 71:
FreeText
C. Page 431/477.
Memorizing these takes a bit of time and chemo-tox man but its fairly easy.
This patient has symptoms of dilated cardiomyopathy caused by the cardiotoxic
doxorubicin.
Shouldve given her concomitant dexrazoxane.

Page 72:
FreeText
D. Page 297/321 FA2019.
The patient clearly has a VSD. Remember my "VAPE later" mnemonic.
Squatting increases pressures in the right heart, sending blood back into the LV and
allowing the
blood to then go into the aorta, fixing the cyanosis.
A VSD would cause a systolic murmur whenever the LV contracts, cuz itll send blood into
the RV.
This is given in the question.

Lung problems are usually a problem related to the left heart.


Pulsating aortic mass would be maybe related to aortic issues not a VSD.

Page 73:
FreeText
F. Page 537/599.
Muscular blockade, leading to fever and muscle rigidity? Malignant hyperthermia.
Classic presentation. Treat with dantrolene.
Stamp
Image

Page 74:
FreeText
D. Page 651/735.
This is math. The graph shows lung capacity / transpulmonary pressure, which is
compliance.
Recall that compliance is volume / pressure.

Choices A and B are irrelevant, and choices C and E go together (and have increased
compliance).
Choice C is the only unique answer where compliance decreases.

Page 75:
FreeText
C. Page 344/376.
Very straightforward answer choice.
The patient has SIADH.
ADH causes anti-diuretic so you retain water.
This means your urine solute concentration (aka urine osmolality) will be higher than your
plasma solute concentration.
The plasma solutes will be diluted in all the water the patient will retain.
The other answer choices all become plausible "maybe" choices if you dont understand
SIADH and the terminology of osmolality very well.

Page 76:
FreeText
D. Page 502/562 FA2019.

https://www.youtube.com/watch?v=A8S3B9p1t_g

This is Wallenberg syndrome.


"Dont PICA (pick a) horse that cannot swallow."
The patient has dyshphagia and some sensory deficits.
The only label pointing at something posterior is D.

Page 77:
FreeText
C. Page 365/406.
Somatostatin has an analog known as octreotide.
The other hormones listed dont really have a commonly tested analog.
Somatostatin causes static stasis of all GIT activity and can treat acromegaly, carcinoid,
and variceal bleeding.

Page 78:
FreeText
D. Page 331/361.
This kind of presentation of abnormal T4 concentrations where a patient is presenting with
labs or symptoms that seem like a thyroid
imbalance is special in cases of pregnancy, OCP use, hepatic falure, steroid use, or
nephrotic syndrome.

This is basically a question of thyroid binding globulin, which is high in pregnancy and OCP
use (estrogen increases TBG production.
The globulin is holding on to all the thyroid, creating a scenario of hypothyroidism so the
body has more TSH formation but inactive T3 and T4.

The other answer choices are irrelevant.

Page 79:
FreeText
D. Page 729 of my PDF. Pg 164/169 and pg 631/710.
This is a recurring questions NBME loves asking about.
These warty lesions on the true vocal cords are a result of HPV 1, 2, 6, 11.
Stamp
Image
Stamp
Image

Page 80:
FreeText
A. Page 310/338.
If a patient presents like this, it is important to know what tests to order even for CS and of
course in real life.
This is a patient with headaches and jaw pain, which should immediately ring a bell for "jaw
claudication" of giant cell temporal arteritis.
This patient should be started on prednisone asap before results of ESR return, because
this can lead to irreversible blindness via opthalmic artery occlusion.

Page 81:
FreeText
A. Page 391/435.
Another excellent broad presentation. Any man with diarrhea without a fever is having GIT
issues unrelated to infection.
On top of that, it is fatty diarrhea, indicative of malabsorption. Bloody diarrhea (dysentery)
would be indicative of inflammation.
The alcohol dependence and abdominal calcification are indicative of pancreatitis, which
seals the deal.

Page 82:
FreeText
E. Page 425/470.
The patient had a strong trauma from the horse kick, which is no joke.
Myelofibrosis is from the hematology chapter, which causes the bone marrow to be
fibrosed due to increased fibroblast activity.
These patients have splenomegaly.
Trauma plus splenomegaly is an easy stage for splenic rupture, leading to nothing more
than anemia (normal leukocyte and platelet count).

The other answer choices are irrelevant.


Mesentaric thrombosis would cause extreme bleeding inside and the patient would go into
hypovolemic shock. The same for choice C.
Choice B can be seen in alcoholics and he might be an alcoholic but that is not what is the
core of this question.
Choice D has nothing to do with his upper left quadrant.

Page 83:
Square
Rectangle
Line

Line

FreeText
B
FreeText
B+
FreeText
B
FreeText
B0
FreeText
MAN
FreeText
WOMAN
FreeText
BB = 100%
beta globin
FreeText
BB0 = 50%
beta globin
FreeText
BB+ = 75%
beta globin
FreeText
B0B+ = 25%
beta globin
FreeText
C. Page 410/454.
Pretty clever question for a topic that is not really inherited in a standard pattern.
The punette square makes it very simple.
You can cancel out choices D and E because for D, with 50% beta globin you will need
transfusions, and choice E says half the kids will be 100%,
which is unlikely given that the father has a null mutation and the mother has mutation too,
so then only a quarter of kids will be 100%.

Not an easy or straightforward question unless you are confident and comfy with genetics
and think of using one of the few genetics tools, the square.

Page 84:
FreeText
C. Page 161/166.
This is clearly lice related problems that commonly spread in early school years.
Very easy straightforward question.

Page 85:
FreeText
A. Page 444/495 FA2019.
I always remember the uworld colored images of sensory innervation and that helps.
Obturator is an adductor nerve.
It innervates adductor longus brevis and magnus, obturator externus, and the gracilis &
pectinues.

The obliques are not hip muscles or even part of the legs. The internal oblique is
innervated by iliohypogastric.
Iliopsoas has an iliacus portion innervated by the femoral nerve.
Obturator internus was a trick answer choice you must remember is something that is
supplied by
the nerve to the obturator internus, a branch directly off the sacral plexus.
The piriformis also has a nerve to the piriformis from the sacral plexus.
Stamp
Image
Stamp
Image

Page 86:
FreeText
C. Page 191/201 and 195/205.
Very easy question. Ciprofloxocin is a fluoroquinolone, which works on topoisomerase 2
and 4.
Topoisomerase 2 is DNA gyrase. Just pure memorization.

Choices A and B are very unlikely as cultures only showed E coli, so there is no other
bacteria that could conjugate and send that plasmid.
Ciprofloxacin acetylase is a real mechanism of E coli resistance to -floxacins.
But the 23s ribosomal RNA Methylase is irrelevant. That would be for macrolide resistance.
The 50s subunit includes the 5s and 23s subunits.

Choices D and F are completely off.


Choice E is related to aminoglycosides.

Page 88:
FreeText
D. Page 157/162.
Source of image --> http://spot.pcc.edu/~jvolpe/b/bi234/lec/2_parasites/images/
P._vivax.htm
This is Schuffner stippling due to Plasmodium vivax.
In general you see something like this you immediately think of Plasmodium.
It could be Chloroquine resistant (an answer choice that should make you think of
Plasmodium), but not necessarily.
He must have had hypnozoites before he came to the US a year ago.

Page 89:
FreeText
B. Page 553/617 of FA2019.
Very nice question.
Her misery after the breakup makes you think of dependent.
Her comment about only being able to trust the doctor and no one else makes you think of
paranoid.
Remember the clusters to help figure this out.

She best fits borderline due to her unstable mood, impulsive reckless drinking, emotional
emptiness, and work related splitting.
Antisocial is in the same cluster B but that would be criminality.
Narcissistic is also a cluster B but that would be grandiosity that she does not exhibit at all.

Page 90:
FreeText
E.
This question was hard.
They try to present the question as a simple diabetic neuropathy question.
The allodynia (pain from things that do not normally cause pain) is key here. This patient
has increased sensitivity.
Allodynia is not due to central things such as the DRG, GABA of the dorsal horn, receptors
in the thalamus, or dorsal root afferents.
Choice E depicts a peripheral receptor being overactivated, as is the case in peripheral
sensitization due to the diabetes.

Page 91:
FreeText
A. Page 300/325.
Classic presentation of an elderly (possibly white man) with an extensive smoking history;
AAA.
Atherosclerosis most commonly occurs in the abdominal aorta. This inflammatory process
leads to aneurysmal dilatation of the abdominal aorta.
In the image, calcification of the aortic wall has occurred.

Good knowledge -- http://www.lumen.luc.edu/lumen/MedEd/Radio/curriculum/Surgery/


aneurysm2.htm
Amazing case scenario -- https://www.ncbi.nlm.nih.gov/pmc/articles/PMC6178120/

Page 92:
FreeText
C. Page 442/488 of FA2019 but this question's answer is not really in FA.

The flexor digitorum profundus has a lateral median nerve half and a medial ulnar nerve
half. It inserts at the DIP. Profundus is profoundly long.
DIP - P for profundus.
This patient's lateral index finger is injured so thats a laceration of the median nerve.

Choice D. The lumbricals flex at the MCP joint and extend the PIP & DIP joints. Lateral
lumbricals by median n and medial lumbricals by ulnar n.
Lumbricals lengthen.

Choice E.The flexor digitorum superficialis is innervated by the median nerve and it inserts
at the middle phalanges of the medial 4 fingers.
This helps flex at the PIP.
Stamp
Image
Stamp
Image
Stamp
Image

Page 93:
FreeText
C. Page 642/723.
This is literally a medication used for abortion.

Page 94:
FreeText
B. Page 307/335.
The severe chest pain might be indicative of a potential MI or HF.
The question gives everything needed to make the right choice but it is based on your
knowledge of cardiovascular topics.
The clammy, sweating patient with normal temp but low BP seems like a case of shock.
More specifically, this must be cardiogenic shock (not septic because of normal temp, no
infection).

Since the patient has crackles in the lung, blood must be backing up and pressures must
be high in PCWP. This immediately brings you to A & B.
However, PCWP can be high or low in cardiogenic or obstructive shock based off of the
specific issue causing the shock.

Recall that the only shock where SVR is low is distributive shock, so you know for SURE
that SVR must be high.
I speak in my book about how no matter what shock we discuss, the underlying physiology
is always based off of CO x SVR = MAP.
In distributive, SVR is very low so CO tries to be increased to maintain MAP.
In other shocks, SVR is high and CO is low, which brings us to choices B and D. 50%
chance of guessing right.

Figuring out whether PCWP would be high or low comes from what i initially said, the
normal crackles in the lung.
This patients JVP is high due to poor forward-flow of blood in this patients specific case. So
their PCWP must be high too for the same poor forward-flow.
If the infarct were only on the right heart, blood would back up and the JVP would be even
higher and not much blood would make it to the left
heart so the PCWP would be low in that case. But most cardiogenic shock cases have
infarct of both sides of the heart.

Page 95:
FreeText
E. Page 318/347 FA2019.
Memorization of the antiarrhythmics takes time and practice.
The class 3 AIDS drugs are potassium channel blockers. Amiodarone is very high-yield.
I once saw a UW question that tested on ONE SPECIFIC thing. As bad as amiodarone's
side effects might be, it does not cause torsades.
The other drugs, Sotalol and Ibuitlide cause torsades (QT prolongation).
On top of that, sotalol also has beta blocker activity that reduced the patients pulse and BP.

Adenosine is a high-yield cool medication i review extensively in my book.


Flecainide (eat more fries please) is one of the class 1c meds. In class 1, only thet class 1a
cause torsades.
Metoprolol is just a beta blocker, so that is irrelevant.
Phenytoin can be placed in class 1b (lettuce mayo tomato please).

Page 96:
FreeText
E. Page 122 for TNF, and page 140/141 of FA2019 for TB.
First of all, choices A-D make no sense.
Second, remember your TNF alpha inhibitors. This patient started infliximab, which is one
of the "In Ada Ce Goli" meds from the Hindi mnemonic I made.
The mnemonic says "In Ada Ce Goli marti, totally not fair." It is a corny phrase from an old
Indian song. It means you're shooting at me with your beauty,
totally not fair. Something like that but it gets the job done. Totally Not Fair is for TNF.
The patient must have latent TB that was previously locked in the granuloma. With the
TNFa inhibitor, that granuloma got weak and the TB reactivated.
Now she has Potts. Disseminated TB causing osteomyelitis. And TB, Mycoplasma, is acid
fast so it does not gram stain.

Page 97:
FreeText
B. Page 400/444 for eosinophils, and page
Great question. Tough choices.
Read the question carefully. They want the result of this reaction.
The reaction itself is initiated by mast cell degranulation (allergies).
Mast cells release many things and the histamine and eosinophil chemotactic factors are
crucial.

The result is from later activity by eosinophils, which are activated by IL5. There is IgE
clustering and cross linking.
Type 1 HSR.

Page 98:
FreeText
D. Page 576/645.
Losartan is an ARB.
This is given instead of an ACEinhibitor if there is a chance the patient might get bradykinin
induced cough. Bradykinin can be high with ACEinhib use.

Normally, RAAS causes eventual angiotensin 2 production which works to cause


hypertension.
Without angiotensin 2 activity, there will be a feedback that will cause more renin to be
made (A is wrong) and more angiotensin 1 and 2 to be made.
There will not be more aldosterone made.

Angiotensin 2 uses AT1 receptors to induce more aldosterone secretion from zona
glomerulosa, but this receptor will ALSO be blocked.
If there was more aldosterone, the patient would still have hypertension with losartan
treatment so C is ridiculous.

Angiotensin 2 is the final messenger that causes all the effects, and it will not be able to
anymore.

Page 99:
FreeText
D. Page 479/537 of FA2019.
Witty NBME wording. The tuft of hair and lack of spinous processes is diagnostic of SBO.

Patient has Spina bifida occulta which is a neural tube defect (failure of fusion of the
neuropores).
Sclerotomes are the part of each somite in a vertebrate embryo giving rise to bone or other
skeletal tissue.
Since a part of this patient's spina bifida included "absense of spinous processes" then a
sclerotome was involved.
Knowing that neural tube defects are an issue with fusion should be enough to get to the
right answer.
Choices A and B are nonsense because of the notochord or neural tube did not develop/
form, the entire CNS would not form.
The yolk sac has nothing to do with this scenario.
Neural crest migration is a bait but it is irrelevant to neural tube defects.

Sclerotomes -> vertebrae, rib cage, occipital bone (part)


Myotomes -> skeletal muscle of the back, ribs, limbs
Syndetomes -> cartilage, tendons
Dermatomes -> skin of the back

Page 100:
Stamp
Image
Stamp
Image
FreeText
A. Page 322/351.
The patient has a failed migration of the thyroid gland?
Not sure why symptoms would appear after age 30.
Either way, the other choices are far from related to the thyroid.

Page 101:
FreeText
2yrs --> 6.7%
3yrs --> 4.5%
4yrs --> 5.6%
5yrs --> 5.2%
FreeText
B.
Just simple math. Nothing insane. They didnt even try to confuse you with incidence vs
prevalence.

Page 102:
FreeText
E. Page 40
Tricky question where many will fall for choice D.
There are generally to types of radiation.
Radiotherapy is ionizing and causes free radical formation.
The non-ionizing UVB from sunlight causes pyrimidine dimer formation that requires
nucleotide excision repair.
Thing NERD- Nucletide Excision Repair of Dimers.

Base excision repair is for non-bulky DNA base alterations, while mismatch repair is for
Lynch syndrome.

Page 103:
FreeText
A. Page 661/754 while page 655/742 discusses A-a.
The patient simply has pulmonary fibrosis, which can often be idiopathic.
This is a restrictive lung disease so all lung volumes generally decrease, making C-F all
wrong.
Choice B is a trick answer. The diffusion capacity of of CO would be LOW not increased.
DLCO is low in interstitial lung diseases (all restrictive lung diseases that are not due to
poor breathing mechanics) and emphysema.
DLCO is higher sometimes in asthma.

Not a hard question at all.


The A-a gradient increases in interstitial restrictive lung diseases due to more oxygen being
in the alveoli but faulty diffusion into arteries.

Page 104:
FreeText
D. Page 322/351.
The patient's mass is a midline mass that is below the hyoid bone.
This most likely is not thyroid tissue since the question is not hinting anything about it.
So then it must be a thyroglossal duct cyst (based on the presentation).
This mass would move with swallowing (compared to a pharyngeal cleft cyst, not
pharyngeal arch like choice A).

The ectopic thyroid tissue would be an answer choice of thyroid, which is not given.
The thyroglossal duct cyst is a derivative of the tongue, which is why it moves.

Good osmosis video -- https://www.youtube.com/watch?


v=snwEs0u99sY&feature=youtu.be

Page 105:
FreeText
B. Page 191/201.
Aminoglycosides have a risk for nephrotoxicity, neuromuscular
blockade, ototoxicity, and teratogenicity.

The nephrotoxicity targets the PCT.


Nephrotoxic drugs cause acute tubular necrosis, which is an
ischemic event. The two portions of a nephron that are
susceptible to ischemia are the PCT and the thick
ascending limb (not labelled).

Page 106:
FreeText
D. Page 302/327 of FA2019.
This question is about the evolution of an MI.
It has been 18 days since the patient had his MI.
Memorizing this evolution takes revision and time, as usual. But it is important to
understand that fibrosis takes lots of time.
The contracted, type 1 collagen, fibrotic scar tissue forms after a month.

Page 107:
FreeText
E. Page 233/243.

Administered 80mg dose


Distribution half life = 3min
Elimination half life = 24 hours
30 minutes have passed and serum has 4ug/mL.

This question gave so much information. Intimidation factor.


Just read the question. They just want volume of distritbution, which is drug administered
divided by plasma drug concentration.

So that would be 80mg and 4ug/mL. Now converting it becomes a thing.


(4 micrograms /mL)(1000mL/L)(1g/1million micrograms) = .004g/L
(80mg)(1g/1000mg)=.08g
So .08 / .004 = 20L

Page 108:
FreeText
D. Page 497/555 and pg 502/562 of FA2019.
The patient is presenting with PICA infarct (same as q76, q 26 of block 2).
Since the patient lost sensation on the right face, the lesion must be on the right so that
knocks out half the answers.
Now you are left with B, D, F, and H.
The first losses the patient has are mainly pain and temperature, meaning the
spinothalamic nucleus is involved.
The loss of his vocal cords and gag reflex are indiciative of CN9 and 10, which are on the
medulla.
This brings you to just choices B or D.
You pick dorsolateral if you studied the anatomy of the images below.
The spinothalamic tract runs dorsolaterally.
Stamp
Image
Stamp
Image

Page 109:
FreeText
B. Page 264/276.
This is a question of autonomy. We must respect the patients choice to not accept medical
care.
However, he does not seem psychiatrically stable as he mentioned he has an invention that
will cure arthritis.

Ambulatory administration of antibiotics would be forcing treatment he does not want.


Home-visit care is also something he has not asked for.
Transferring to the psych outpatient facility is unnecessary and extra.
Psych transfer is rarely dont when a patient is extremely depressed, suicidal, or expressing
schizophrenic/mood disorders.

Page 110:
FreeText
F. Page 268/280.

VERY tricky question that truly tests your understanding of deriving information from each
sentence.
The patient is doing well! No diabetes, no atherosclerosis or other arterial inflammation,
and no concerns.
The patient has a very minimal increase in his PSA and some growth of his prostate, but he
has not
complained about issues regarding urination. It definitely is not prostate cancer either (no
nodules).

A- False. Dorsalis pedis and pulses are normal.


B- Possibly but not the cause of his sexual dysfunction.
C- False, not diabetic.
D- False, no nodules or metastatic symptoms.
E- False, he is having erections and having sex, but he just does not find his orgasms
pleasing.
F- Correct. Normal aging causes men to have a weaker ejaculation and longer refractory
period.

Page 111:
FreeText
C. Page 269/281.
This question is about primary prevention.
Examples include health promotion through vaccinations, exercise, diet, and smoking
cessation.
Women ages 30-40 need to invest their time into weight-bearing exercises so that they can
prepare for the skeletal issues revolving around menopause.

B and C would be relevant to an overweight group but that is irrelevant here.


A would be for groups known to fall into alcoholism.
D is critical for heavily sexually active teens. On top of that, "testing" for disease is a form of
screening, which is for secondary prevention.
The same goes for annual mammography, which is a screening for breast cancer.

Page 112:
FreeText
D. Page 625/703. Sexual differentiation on page 608/684.
This patient most likely has the X-linked recessive androgen insensitivity syndrome (AIS).
This patient is probably a male (46XY) with a disorder of sexual development.
The blind ending vagina means that the lower vagina (external) developed, but the upper
vagina (internal) did not.
The upper vagina develops from the Mullerian duct in the absence of MIF. This patient
most likely has testes (in the labias).
The testes make MIF and cause the mullerian duct to disintegrate.
Testosterone, estrogen, and LH will be high.

The factor differentiating this from mullerian agenesis is the lack of pubic and axillary hair in
AIS.
Stamp
Image

Page 113:
FreeText
D. Page 554/618.
Awkward right? I always think of this as "She is making it a fact that she has a disorder."
Factitious disorder. Symptoms are intentional, motivation is unconscious. The patient just
wants to be sick and get sympathy.

Page 114:
FreeText
A. Page 298/323 for fibromuscular dysplasia and 2 pages down discusses atherosclerosis.
There is a lot to work with in this question as well.
She has bilateral proximal renal artery stenosis.
You can cancel out choices B D and E. B is unreal. D and E affect large proximal vessels of
the ascending aorta.

Atherosclerosis mainly affects the PROXIMAL 1/3 of renal arteries and is the #1 cause of
renal artery stenosis.
Fibromuscular dysplasia affects the DISTAL 2/3 of renal arteries and is the #2 cause of
renal artery stenosis.
This patient was fine in her previous visit 1 year ago, which also begs to think that this year
she might have developed atherosclerosis.
Fibromuscular dysplasia usually occurs in women of child-bearing age and this patient is
well into menopause.

Page 115:
FreeText
E.
Very nice question with a good take home lesson. Dosage matters.
You can remove ketorolac (an NSAID) and penicillin (an antibiotic) from the choices.
Ketorolac would help but help as it is a good NSAID and salicylate for acute gout but must
be given in high dose for urinary excretion of urate.
Same for choice A, they didnt say high or low dose so that makes it hard.

Probenecid is a medication for chronic gout and it inhibits reabsorption of uric acid in the
PCT so its perfect for what theyre asking for.
The question mentions his current preventive therapy is not working (maybe hes on
allopurinol).
So this would be his new preventive therapy.

Page 116:
FreeText
D. Page 473/531 of FA2019.

Recurring theme. NBME has asked multiple times about melanoma occurring in the palms/
soles of AA patients.

https://www.aimatmelanoma.org/melanoma-risk-factors/melanoma-in-people-of-color/acral-
lentiginous-melanoma-alm/
Acral lentiginous melanoma is the melanoma on the palms, soles, and nail beds.
It is uncommon but it is the most common melanoma in people of darker skin or Asian
descent.

Melanin in darkly pigmented skin can provide an SPF (sun protective factor) of 1.5 - 4.
Amount of UV light getting through is 1/SPF, so a SPF of 4 reduces UV radiation by 75%.
So the areas without this protective feature are most at risk (palms/soles).

Page 117:
FreeText
Page 121.

Question 7 block 2 here was also on this same topic and had the same answer.
GMCSF will help with the bone marrow suppression.

For revision, you should know exactly what the rest of these are for.

INFa is used for chronic hep C, hep B, and RCC. (Think abc arcc).
INFb is used for multiple sclerosis while INFg is used for CGD and IL12 receptor deficiency.

IFNg and IL12 cause differentiation into Th1 cells.


IFNg also stimulates macrophages.

IL2 and IL4 cause differentiation into TH2 cells. (page 101).
IL4 also causes class switching to IgE and IgG.
IL6 is for acute phase reactants (Hot T Bone stEAK).

TGFb and IL10 attenuate the immune response and work with Tregs.

TNFa is for cachexia and maintenance of granulomas.

Page 118:
FreeText
A. Page 367/408.
NBME trash strikes again.
So the duodenum is where pancreatic proteaseses like trypsin and chymotrypsin break
down proteins to peptides.
These peptides get further broken down into amino acids in the brush border of the
intestines
Stamp
Image

Page 119:
FreeText
D. Page 517-518/577 discusses motor neuron signs and spinal lesions.
This patient has fasciculations, lower extremity weakness, and atrophy. Sensory stuff is
fine. These are signs of LMN lesions.
This could be ALS but the question is not asking for a specific diagnosis.
This question does a good job testing what you understand about various parts of the CNS.

Choice A-- The cerebral white matter is full of myelin (lipid) and forms the deep parts of the
brain in between
grey matter such as the basal ganglia and brainstem nuclei.
It serves to help coordinate communication between different brain regions. Multiple
sclerosis affects white matter.

Choice B-- In contrast, gray matter comprises the main components of the brain (pinkish
due to capillaries).
Composed of neuronal cell bodies, dendrites, glial cells, and synapses.
The anterior grey column is for motor neurons (pyramidal tract so corticospinal +
corticobulbar),
while the posterior grey column is for sensory neurons (dorsal and spinothalamic).
C-- The internal capsule is a high-yield part of the brain. It is related to UMN lesions (q on
my pdf page 577).

D-- Peripheral nerves would have sensory involvement as well.


Stamp
Image

Page 120:
FreeText
F. Page 459/517.
This is like looking at a black and white image of the urate of gout.
You must remember that the crystals in pseudogout are rhomboid.
Plus the question explained the tophi this patient is getting, which further indicates gout.

Needle shaped crystals = gout = urate. Done.

Page 121:
FreeText
D. Page 175/185.
First thing is that the patient has an infection. It seems like a very severe lung infection for
the past 2 days.
The most notable thing is his leukocyte count is low and the breakdown is not distributed
uniformly.
90% neutrophils but he seems to have lost lymphocytes and monocytes. 5% of 2000 is
100. Very low CD4 count.
Could even be pneumocystis jirovecii.
The HepC is related to IV drug abuse, which can also lead to HIV infection.

Page 122:
Stamp
Image
FreeText
D. Pg 376/418 for Crohn and page 409/453 for heme.
Crohns disease has many issues (ulcers, fistulas, bleeding) but it
does not cause IDA. UC has bloody diarrhea and also rarely has
IDA but it is possible.

The peripheral smear is showing megaloblastic anemia.


There are many macrocytic RBC with pale centers.

Choices B and C are irrelevant.

Choice D, the correct answer, is explained by the pathway here.


Vitamin B12 is necessary for convertiing 5M-THF into THF, the
active form. Without B12 this conversion will not occur and you
will not get THF to be able to make more DNA. So this patient
must have B12 deficiency due to involvement of injury to the
terminal ileum, which is common in Crohn.

Page 123:
FreeText
D. Page 611/687.
This is a very easy question. There are two cases that can occur, an oophorectomy or a
hysterectomy.
During an oophorectomy (as in our patient), the suspensory infundibulopelvic ligament that
suspends the ovarian vessels is ligated.
During a hysterectomy, the cardinal transverse cervical ligament holding the uterine
vessels is ligated.

Both cases carry a risk of ureteral injury that often presents with unilateral flank pain (if
bilateral, patient cant void either).

Page 124:
FreeText
B. Page 665/761.
The patient fits the presentation of someone who has sleep apnea.
I always think of sleep apnea as "you arent even breathing." This is one reason why this is
under respiratory, because it causes hypoxia.
Try snoring for an hour straight while youre up and about on your day.
You will run out of breath and get weak and tired. Apnea is a real issue. Patients just dont
notice because theyre asleep.

Page 125:
FreeText
E. Page 70 FA2019.
First of all. WTF is plantar dorsiflexion? NBME Trash.

This patient has neurological deficits but without megaloblastic macrocytic anemia.
This patient has anemia that is most likely due to hemolysis.
Vitamin E is an antioxidant that protects RBCs from free radical damage.
Deficiency can lead to hemolytic anemia and demyelination of the posterior columns and
spinocerebellar tract.

Often confused with vitamin b12 deficiency which is similar in presentation.


Deficiency of b12 leads to macrocytic megaloblastic anemia along with subacute combined
degeneration
involving the lateral corticospinal tract as well.

Page 126:
FreeText
B. Ethics
This is just typical gynecomastia due to puberty related spikes in testosterone and
estrogen.
Choice A is wrong since that is not when puberty occurs.
Choice C is unnecessary.
Choice D is not something you can tell someone. People will get comfortable when they get
comfortable. Some never do.
Choice E is a lie lol.

Although I must admit I would hate to tell a little boy hes gonna have big boobs for another
year and a half...

Page 127:
FreeText
B. Page 170/179.
The patient has AIDS which is important because this patient cannot be immunized, so
they are susceptible to everything.
The cousin is unimmunized (maybe theyre from a 3rd world country) so this cousin can
have any disease (MMR, TDaP, etc).
The koplik spots are practically pathognomonic for measles (a paramyxovirus, remember
sketchy's paranormal mixer).

Page 128:
FreeText
A. Page 264/276.
This patient is in need of this life saving surgery.
Beneficence is acting in the benefit of the patient.

Common good and integrity are just filler answer choices.


Autonomy involves listening to the patient.
Nonmaleficence involves not hurting the patient. An example for this scenario would be to
NOT do a surgery that might possibly kill the patient.

Page 129:
FreeText
E. Page 206/216.
They really tried to make the answer choices come off extremely intimidating.
But overall its clear that the patient has damage from the stressor; 40 years of extensive
smoking.
Metaplasia is a normal response to stressors. The mass in the lobe of the right lung could
be from the smoking.
This patient might even have GERD related to his obesity.

Regardless, they said he has squamous metaplasia which means that the ciliated columnar
epithelium is becoming stratified sqamous epithelium.
The columnar cells are normal, and they get replaced by normal looking squamous cells.
The fact that this is happening is abnormal but it is reversible.
Good question to test on the understanding of pathology.

Page 130:
FreeText
B. Page 639/720.
Very nice question. The patient has urinary incontinence (dribbling of urine), prostatitis
(tender prostate), and might even have a fever.
E coli is the #1 cause of a UTI but this question is about more than that.
E coli is also the #1 cause of acute bacterial prostatitis in older men (younger men would
be C trachomatis or N gonorrhoeae along with sexual activity).

Dont think of picking C trachomatis or N gonorrhoeae without a urine culture being


mentioned or something

Page 131:
FreeText
B. Page 688 of my annotated FA.
This is a recurring theme in NBME.
This patient has a rectocele. The wall of the rectum is bulging towards the vagina.

When a rectocele becomes large, stool can become trapped within it, making it difficult to
have a bowel
movement or creating a sensation of incomplete evacuation. Symptoms are usually due to
stool trapping,
difficulty passing stool, and protrusion of the back of the vagina through the vaginal
opening.
During bowel movements, women with large, symptomatic rectoceles may describe the
need to put their
fingers into their vagina and push back toward the rectum to allow the stool to pass.
Rectoceles are more common in women who have delivered children vaginally.
Stamp
Image
Stamp
Image
Stamp
Rectocele.

Page 132:
FreeText
C. Page 495/553 FA2019.
This patient has temporomandibular joint dysfunction, which is not a topic in FA.
However, this page does mention the muscles of mastication.
The other muscles listed are irrelevant and you should have a decent idea of where they
are.

The levator veli palatini elevates the soft palate during swallowing to prevent food from
entering the nasopharynx.
Stamp
Image
Stamp
Image
Stamp
Image
Stamp
Image

Page 133:
FreeText
D. Page 47 FA2019.
When you first see this question it should NOT ring a bell for a lysosomal storage disorder.
The LSDs have neurological symptoms.
All the LSDs have involvement of other organs, histological findings, and involvement of
many pathologies other than hepatic problems.
The lipid-filled hepatocytes were a bait to make you think of "lipid-laden macrophages" of
Gaucher and Niemann Pick disease.

This patient has Zellweger syndrome, an autosomal recessive peroxisomal disorder due to
mutated PEX genes critical for peroxisome function.
The presence of VLCFA is an indication of failure of beta oxidation, one of the key functions
of peroxisomes.
Patients will have elevated VLCFAs and there is no treatment so death is imminent by age
of 1 year.

Page 134:
FreeText
D. Page 558/623.
This patient is presenting with opioid withdrawal.
The fact that heroin is an opioid drug made from morphine is something that catches IMGs
off-gaurd.
The "piloerection" is the giveaway for opioid withdrawal (cold turkey).
Patients present with flu-like symptoms.

Mnemonic.
Opioids are for peace and euphoria.
Opioid intoxication puts you to sleep, so its dark, and pupils constrict.
Opioid withdrawal puts you in pain, so its fight/flight mode, and pupils dilate.

Page 135:
FreeText
E. Page 240/250.
Very nice and fair question! The patient has postpartum urinary retention. The increased
progesterone can inhibit bladder muscle activity.
Bethanechol is a cholinomimetic that acts on M3 receptors to activate the bladder smooth
muscle.
The M3 receptor also causes bronchoconstriction, which would be bad for the patient's
asthma.

The other answer choices are not related to bethanechol.

Page 136:
FreeText
A. Page 571/638 of FA2019.
This question is also very straightforward as long as you have a strong understanding of
renal physiology.
The image on this page is critical. The thickness of the arrows pointing to or from the
glomerulus determines their intensity.
Pgc is the strongest. It is the glomerular capillary hydrostatic pressure, which pushes fluid
into the bowman space.
Pbs is the 2nd strongest, which pushes fluid into the glomerulus (our answer).

The oncotic pressures are weaker.


PIgc is the oncotic pressure inside the glomerulus and it pulls fluid from the bowman space.
PIbs is the weakest and it pulls fluid into the bowman space.

Our patient has a hydronephrotic kidney and a dilated ureter. This indicates there is
blockade so urine is backing up.
Thus there is a ton of pressure up the ureter all the way up the collecting tubules and into
the bowman's space!
Pretty simple and clear. This is the opposite flow of filtration so GFR decreases.
Stamp
Image
Ink
Drawing, 2 Lines
Ink
Drawing, 1 Line

Page 137:
FreeText
B. Page 474/532.
ALERT- NBME Trash.
So you can definitely cancel out C because that is irrelevant to aspirin.
Choice E is also irrelevant to aspirin's mechanism. Same for choice A, aspirin does not
break VEGF.
This leaves choices B and D.
Never heard of thrombosthenin so I chose B and this is BULLSHIT because platelet
adherence and aggregation are DIFFERENT things.

Aspirin inhibits platelet aggregation and produces a mild bleeding defect by inhibiting COX
enzymes.
Loss of TXAs messes up platelet aggregation. Not adherence. Get your shit straight
NBME.

Also, for completeness, thrombasthenin is just a contractile protein inside platelets.

Page 138:
FreeText
A. Page 238/248.
Intricate question. I dont agree on the way this is being tested and it is quite unreal.
Not the way this will be tested on STEP 1.

Regardless, you should be able to cancel out B-E and then find some way A is even
possible.
This depends on how much youve been studying and how close you are to taking step 1.

I discussed the g-protein linked second messengers and receptors in pharmacology a lot
and ive always advised to revisit often.
This patient has alcohol withdrawal.
He had a seizure, which must have lead to an increase in catecholamines.
The catecholamines increased his BP.
His serum K is low but his urine K is normal, so the K must have shifted into cells.
Insulin is notorious for intracellular shift of K. So what happened here?

Well, beta 2 receptors increase insulin release. M3 also increases insulin release.
And alpha 2, the receptor that decreases everything, decreases insulin release.

Page 139:
FreeText
A. Page 669/768.
You will only miss this question if you are very new to step prep or have not revised
respiratory and done UW.
The lung cancers are notorious for ectopic secretion, notably small cell (ACTH, ADH, and
antibodies causing lambert eaton and more).
This patient must be secreting ACTH, which ramps up cortisol release and can also lead to
more insulin release (which caused the hypokalemia).
Page 140:
FreeText
F. Page 492/550.
These images are never perfect enough.
A = CN III
B = CN V
C = CN VI
D = CN VII
E = CN VIII
F = CN X

Remember the rule of 4's.


1-4 in midbrain
5-8 in pons
9-12 in medulla.
And 3, 4, 6 ,12 are medial.

Our patient's difficulty swallowing and hoarseness


are indicative of CN10 vagus injury, so that must
be F since it is the only one coming out of the
medulla. It also has a thick trunk since its big.

They have used this same image repeatedly.


Stamp
Brain stem anterior view.

Page 141:
FreeText
D. Not in FA.
I had never heard of peptide transporter TAP.
I do know adenosine deaminase deficiency can cause SCID, which is irrelevant here.
Bare lymphocyte syndrome is a form of SCID, but the mechanism is different.
Fas ligand is related to apoptosis (pg 208/218 fa2019) and is of the TNF family, so that is
irrelevant here.
And IL2 is for making Th2 cells and that is irrelevant here.
So I just picked D.

TAP deficiency is the BLS1. Symptoms can include recurrent bacterial infections of the
respiratory tract and chronic skin lesions.
Why you would know this is beyond me. It is not in FA.

This is how NBME induces crippling depression in medical students who pay for an exam
that comes with no explanation.

Page 142:
FreeText
E. Page 203/213.
NBME trash alert. All NRTI can cause BMS and they listed two..

HIV therapy is a common NBME topic and will be on your step as well.
The NRTIs have weird names that need to be revised often to memorize.
The special ones are tenofovir (T for nucleoTide, while others are all nucleosides),
zidovudine (can be used in pregnancy), didanosine (pancreatitis), and
abacavir which is contraindicated in HLAB5701 patients.
It was wrong of them to list both zidovudine and lamivudine as answer choices because all
of the NRTIs can cause bone marrow suppression.
However, "Zero bone marrow wiith Zidovudine" is a sketchy memorization mnemonic.

The NNRTIs cause rash and hepatotoxicity and efavirenzzz gives vivid dreams when you
zzz sleep.
Protease inhibitors have various GI side effects and should not be taken with rifampin so
take rifabutin instead if you have TB.
Integrase inhibitors can raise CK. And thats about it overall.

Page 143:
FreeText
G. Page 549/612.
This is an elderly patient. Depression presents differently in the elderly.
This couldve been atypical depression. The two key features are mood reactivity and
leaden paralysis.

He meets the SIGECAPS criteria of MDD.


He has sleep disturbance, anhedonia, and loss of energy. His weight loss could be due to
appetite changes, the A of SIGECAPS.
If both MDD and atypical depression were answer choices, I would pick the MDD since
neither leaden paralysis nor mood reactivity are mentioned.
However, one of the important diagnostic requirements of MDD is a self-reported
depressed mood or anhedonia, and he mentions that.
So MDD is treated with SSRIs and paroxetine is perfect for that.

Amitriptyline - TCA. The C's of side effects. Cardiotoxic. Not safe in elderly according to
Beer's criteria.
Buspirone - "Im anxious I will miss my bus." For GAD.
Carbamazepine - Neuro drug. "CARBs are bad for blood" because of agranulocytosis and
aplastic anemia. Mainly for trigeminal neuralgia.
Haloperidol - Typical antipsychotic, not an antidepressant.

Page 144:
FreeText
D. Page 40.
You should have been able to cut this down to C or D.
Then, realize that the question is not about pyrimidine dimers or xeroderma pigmentosum.
Thus, this must be about x-rays, which cause double-strand breaks.

Simple. Just keep it simple.

Page 145:
FreeText
B. Pg 450/506 and pg 454/511.
Straight up honest description of achondroplasia, like Peter Dinklage (Tyrion) of Game of
Thrones.
The description of a normal trunk and large head relative to body size are key.
The mutation causes an activation of FGFR3, inhibiting chondrocyte proliferation.
The autosomal dominant form is lethal so everyone alive mustve had a sporadic mutation.
One cool fact is that it is associated with a high paternal age.
A-chondro -- Without endochondrol ossification.
Stamp
Image

Page 146:
FreeText
C. Ethics
The third question about autonomy this NBME.
The mentally capable and competent patient does not want to live this meaningless life of
being hooked up.
Plus he has what seems like a terminal disease.

Page 147:
FreeText
D. Page 46 FA2019.
This is a straightforward question about making mitotic cyclins before mitosis occurs.
G2 is before mitosis, so thats it.
G2 is very irrelevant overall and rarely an answer, specially because even in
pharmacotherapy
only bleomycin works at G2 (and topisomerase inhibitors but they also work in S phase).

There are G1/S cyclins that FA discusses and these control the G1 to S phase transition.
Then there are G2/M cyclins which are synthesized at G2 and are responsible for transition
from G2 to M phase.

Page 148:
FreeText
F. Not in FA.
Got it right! So I was initially thinking of the location, inner thigh. Okay. Nothing there but
muscle and skin, etc. No special organs or blood vessels.
Then I thought about the presentation.
Painful and swollen (inflammation) and then circumscribed and very firm (not just firm, but
very, and you know NBME).
M ossificans often occurs after trauma to that area over time. It is a form of metaplasia of
mesenchymal tissue.

AV fistula would not become circumscribed and hard (C&H)


Dermatomyositis does not present like this. pg463/521.
Fascitis is an inflammation, it would not become C&H.
Ganglion would not present with a swollen mass. It could be painful, might become firm
later idk but didnt "feel" right.
Hemangioma are pretty benign and usually just blood vessel related but would not harden.
Sarcoma didnt feel right either.
A synovial cyst would also not harden.
Stamp
Image

Page 149:
FreeText
B. Page 60 FA2019.
NBME trash alert. My NBME trash alert button is about to break with all the trash in this
test.
CF can occur due to God knows how many different problems.
So they did a molecular analysis of the baby using 70 known CF mutations and they only
found a mutation in one allele.
CF is autosomal recessive so the mutation shouldve been on both alleles.
B is the "best" answer choice because there must be some other mutation that wasnt
accounted for and that did not get checked.
However, it couldve also been taht there is some other issue that is messing up something
that interacts with CFTR.

This is actually a fair question because UW also had a very unfair question of this caliber.
Problem is, its ok when UW does it because UW is TEACHING you. Its not ok when a real
test does it, so this is NBME trash.

Page 150:
FreeText
E. Page 433/479.
NBME Trash alert. uff.
I immediately thought of paclitaxel when I saw the given image.
After I didnt see it in the answer choices, the only other explanation was vincristine since
they are listed together.
This is one of the big reasons I tell people to revisit FA for each UW question when you do
qbanks.

Problem is, vincristine INHIBITS MITOTIC SPINDLE FORMATION. But in this image...it
has formed.
They will probably use the same image for another question in some NBME later where
paclitaxel will be the answer.

Page 151:
FreeText
D.
As I said the last time these answer choices popped up in an earlier question. Know your
interleukins and cytokines!
TGFb does not have one specific area it is discussed in the book, but it is mentioned
throughout the book.
Mainly its related to inflammation and fibroblast proliferation.
The other answer choices are really not related to this.

Page 152:
FreeText
B. Page 60.
NBME = Naturally Bad at Making Exams.

In CFTR, the misfolded protein gets retained in the RER and this is in FA.
Cytosol was a trick answer.
UW had some questions regarding this misfolded protein being destroyed by ubiquitination
and proteosomes, but that is different.
Most CFTR misfolds are so misfolded they cannot even leave the RER.
Eventually so much accumulates that the RER practically breaks and then it can be sent to
proteosomes.
Page 153:
FreeText
C. Page 642/723.
Overall, the only forms of contraception youll ever give someone as a gynecologist are
either OCPs or IUDs.
The other things dont require a doctor and are found OTC.
Contraceptive sponge, diaphragm, and rhythm method are really unreal answers.
The first two are devices anyone can buy, and the rhythm method is based off of watching
someones period cycles.
Spermicidal foam is also something you can purchase OTC.

The catch in this question was to use something nonhormonal since hormonal OCPs
caused her migraines to worsen.
A copper IUD is not hormonal and will work great.
Stamp
Image
Stamp
Image
Stamp
Image

Page 154:
FreeText
A.
How appropriate to ask about coronavirus lol.
This virus causes respiratory symptoms and is enveloped.
You know this virus must be enveloped because it does not hold up to acidity or being
dried.
Only coronavirus of all the viruses listed is enveloped, making it the most likely answer
even if no question was posed.
Remember the logic game of "which one of these is least like the other?"

Page 155:
FreeText
A. Not in FA.
Not sure if I wanna hit the "NBME Trash" alert again but it definitely feels like it.
Being in bedrest changes the effects of gravity on your body because you go from an
upright plane to a horizontal plane.
Bedrest over 24 hours eventually causes a shift of fluids away from the legs towards the
abdomen, thorax and head.
This increases venous return to the heart and elevates intracardial pressure.

This increased venous return stretches the RA, releasing ANP, a diuretic that induces urine
output and decreases blood volume.
Normally, the drop in blood volume would be detected by the reduced stretch in the
baroreceptors of the aortic and carotid sinus and induce ADH release.
ADH stimulates the kidney to reabsorb water, reducing urine output and increasing blood
volume.
In supine individuals, the increased venous return also increases stretch of the aortic arch
and carotid sinus baroreceptors, reducing ADH release.
This further increases the volume reduction.
In summary; Supine individuals have increased pressure in the RA and on their
baroreceptors. The RA makes more ANP. The baroreceptors reduce ADH.
Mobile individuals would have reduced stretch of their baroreceptors, increasing ADH and
balancing the volume loss.

Full in-depth conversation here --


https://www.nursingtimes.net/clinical-archive/cardiovascular-clinical-archive/effects-of-
bedrest-1-cardiovascular-respiratory-and-haematological-systems-28-05-2009/

Page 156:
FreeText
A. Ethics.

NBME TRASH ALERT. This question mocks the practice of Islam and practically deems it a
"patriarchal" organization.
Although in some cultures, Islam may be a mockery, it is irrelevant to the exam. Islamic
women are conservative for their connection to Allah.
The presence of her husband will not maintain conservatism.

However, this is probably what the idiot question writer was probably thinking:

Very easy question. Always remain calm and think of what you would have to do as the
doctor and what you would want as that patient.
You are a female Muslim patient who is very conservative (burka). Some such patients are
alright with this kind of problem if their husband is present.
SIMPLY ASKING such a question is PERFECTLY okay. She might say yes, she might say
no. She wont be offended nor will the doctor do anything wrong.

This question is so easy, that if 1000 random non-medical non Muslim people were asked
this, majority would pick A, probably over 80% of them.

Page 157:
Stamp
Image
FreeText
E.
The farmer has tetanus. They put choice A as AchASE because they wanted you to think
farmer-->organic phosphate poisoning
but this patient does not have cholinergic symptoms. Once you recognized the question is
asking about SNARE proteins, you had
to find which of these it could be.

You could cancel out A, C, D, and F since those are irrelevant to tetanus.
That leaves B and E. Acetyl-anything is usually some kind of salicylate and so is this one.
Synaptobrevin (VAMP), SNAP25, and Syntaxin are SNARE proteins involved in tetanus.

Page 158:
FreeText
C. Page 384/427 discusses liver pathology . Page 390/433 discusses cholelithiasis.
Patient has gallstones possibly due to biliary stasis. This has caused damaged to the biliary
tract because a stone must be lodged in the common bile duct.
Now bile will back up the common hepatic duct and cause injury to the epithelial lining of
the biliary tract.
Its leakage into the bloodstream induces the itching shes having.

Any damage to biliary tract epithelia leads to leakage of alkaline phosphatase, which is
significant for cholestasis and its consequences like biliary obstruction.

AST and ALT would be elevated in hepatocyte injury, which is not the case here.
Acid phosphatase is just a phosphatase, so it removes phosphates off other molecules.
LDH converts pyruvate to lactate. Clinically it is significant of hemolysis. LDH is abundant in
RBC.
Bilirubin conjugation occurs in hepatocytes and they are functional so you would not have
an elevation in unconjugated bilirubin.
There would be an elevation in conjugated bilirubin.
Stamp
Image

Page 159:
FreeText
D. Page 261/273.
The data presented is simple.
We are only concerned with the 50-54 age group and they said you can assume a normal
distribution bell curve.
For greater than 296, we are concerned with the people after 1SD because within 1SD
would be people up to 296 mg/dL.

1 SD = 34%
2 SD = 34% + 13%
3 SD = 34% + 13% + 2%

Our patient is the 13 + 2 roughly 15 or 16% based on how you round the decimals. So D.

Page 160:
FreeText
A.
NBME TRASH ALERT. I thought it would be D. And as I checked the answers I was
chanting "dont be NBME trash" as I was concerned about A.
It would be great if the roommate wasnt a peice of shit that smoked inside the house with
other roommates present, but that is not in the doctors control.
The student herself shouldve already asked the roommate to not smoke, or is fully aware of
it and has accepted it. Cant be changed.

Of the remaining choices, getting an air cleaner is the best choice. There are amazing air
purifiers out in the market today that can resolve all of this.

The other things like the dog and her school stress are not new variables.

Page 161:
FreeText
E. Page 487/545.
This patient has dysmetria on the right after a brain injury. Dysmetria is a type of ataxia that
has an
overshoot or undershoot due to a lack of movement coordination. Patients fail the finger-to-
nose test.
This is immediately supposed to make you think of the cerebellum, which comes down to D
and E.
The D might be the flocculonodular lobe or central cerebellum, which would cause
nystagmus and
truncal ataxia. This patient has an issue with the lateral cerebellum causing ipsilateral
dysmetria.
Stamp
Image
Stamp
Image

Page 162:
FreeText
A. Page 302/327 progression of an MI.
This is a high-yield topic that is easy to understand and master.
After an MI, the hypoxic environment of that damaged heart leads to use of anaerobic
respiration and a deficiency of ATP and rise in lactate.
The lack of ATP and rise of lactate is similar to what happens in strenuous exercise. Now,
without the ATP, the Na/K ATPase fails so there
is an accumulation of extracellular K, which predisposes patients to ventricular arrhythmia.

At the same time, there is also an intracellular accumulation of Na and Ca, which pulls in
water and causes edema.
The intracellular Ca also causes hypercontraction of the myofibrils.

After this there is extensive coagulative necrosis and postinfarction fibrinous pericarditis,
granulation tissue that can cause tamponade or MR,
and finally a full scar with a chance of a mural thrombus or HF.

Page 163:
FreeText
B. Page 548/612.
This was a silly question. I approached it using the logic method of "which one of these is
different from the other?"
That being said, DM1 is the only one without a clear pattern of inheritance, similar to
schizophrenia.
I doubt the pedigrees of DM1 and schizo would look similar, but they would be "most"
similar in comparison to the rest.

Page 164:
FreeText
C. Page 392/436.
No reason to measure anything else. You measured gastrin to be high, now discontinue the
PPI and see if the gastrin goes down.
If the patient does not have a gastrinoma or something, then once the PPI is taken away,
gastrin will cause acid release and the gastrin will decrease.
If the gastrin didnt decrease once it successfully lead to more acid production, youd
suspect a gastrinoma of sorts.

Page 165:
Stamp
Image
FreeText
C.Page 514/575.
Giveaway question.
The patient has an acoustic schwannoma.
Schwann cells are neural crest cells.
Stains S-100 positive.

Page 166:
FreeText
A. Page 177/187.
The only AIDS defining illness in this list is EBV lymphoma.
The others can happen to anyone.

Page 167:
FreeText
B. Page 319/348.
Honestly, none of the other choices here have anything to do with this topic.
Adenosine is a very short acting substance that has effects in vasculature.

When the cuff is tight, cells and tissue distal to the cuff will run out of ATP now that new
blood isnt arriving.
There will be an accumulation of ADP (adenosine), which causes vasodilation in vascular
smooth muscle.
Here, adenosine acts via A2 receptors (not alpha 2, but A2), using Gs second messengers
to raise cAMP.

In the heart nodes, adenosine acts via A1 receptors to act through Gi second messengers
and reduce cAMP to lower chronotropy and dromotropy.
Stamp
Adenosine Receptors

Page 168:
FreeText
E. Page 203/213.
Second or third HIV treatment question this NBME. Worth mastering. Theyre not so bad
just takes time and its not complex.
THe NRTIs have many side effects, but the NNRTIs are what are really known for rash and
hepatotoxicity (plus vivid dreams from Efavirenzzz).

The protease inhibitors have metabolic issues for the most part but not really
hepatotoxicity.
Integrase and entry inhibitors are practically clear of any major side effects.

Page 169:
Stamp
Image
FreeText
A.
Why would you know this?
Who truly cares?
Find out in the next episode of more NBME Trash series...
The parotid gland's secretions follow the Stensons duct and pierce through the buccinator
to enter the oral cavity.

Page 170:
FreeText
E. Page 159/164.
This patient has Strongyloides stercoralis infection.
Her recent travel to PNG, cough, alveolar infiltrates, eosinophelia, and stool sample with a
worm are all indicative of intestinal parasite infection.
The physical exam of "cutaneous larva currens" is more specific for S stercoralis infection.
Cutaneous larva migrans is seen more on the feet in patients with Ancylostoma or Necator
infections.

Treatment of Strongyloides is with ivermectin or bendazoles.


Praziquantel is appropriate for worm or liver fluke infections.
Stamp
Image

Page 171:
FreeText
B. Page 307/335.
This patient has cardiogenic shock.
He is most likely experiencing an MI.
Since his heart cannot pump blood well, blood will stay in the heart and increase PCWP
(that knocks out A, C, and E).
To fix this the body will have a sympathetic response and catecholamines will cause
peripheral vasoconstriction (knocks out D and F).
That makes B the answer.

The pulmonary vascular resistance is determined by autoregulation. Since the heart is


failing, it may release local vasodilatory metabolites.
That can cause pulmonary vasculature to dilate.

Page 172:
FreeText
D. Page 117 FA2019. Page 109 discusses respiratory burst.
The patient has chronic granulomatous disease (CGD).
S aureus is catalase positive so it is able to convert its own hydrogen peroxide into oxygen
and water.
Otherwise, patients without NADPH oxidase can still use the bacteria's hydrogen peroxide
to create ROS and kill the bacteria.

For this reason, patients with CGD are susceptible to catalase positive infections.

Page 173:
Stamp
Image
FreeText
E.
Honestly was not 100% sure, but after knowing how the idiots of NBME write their answer
choices, I felt E.
LH would increase testosterone production, which is the true cause of his symptoms.
The symptoms are not bad either, it just looks like hes hitting puberty and its not coming
slow.

Page 174:
FreeText
E. Page 51 FA2019.
Rumor has it this image was taken with a Nokia. That's why the image is still around even
though we have 108mp cameras.

This image is supposed to be showing you blue sclerae. I for one can even argue that I see
white keratin deposition from vitamin A deficiency.

Scars are type one collagen and this is a kid with Osteogenesis imperfecta. Dont always
rely on images when it comes to the NBME.
They might use this same image later to tell you a kid looked at you like this after you told
him what to do, and the answer might be oppositional
defiant disorder for a psych question. So use the facts of the question itself.

Page 175:
FreeText
A. Page 417/461.
The question asks what the bone marrow will synthesize.
Bilirubin is the liver, EPO would be increased in this patient but that is the kidney, ferritin is
irrelevant, and uric acid is also irrelevant.
Nor are ferriting and uric acid made by bone marrow.

Blood loss requires hematopoiesis to make up for the loss.


The first step of heme synthesis is the fusion of glycine and succinyl CoA, which can form
dALA in the bone marrow.

Page 176:
FreeText
E. Page 83.
The child has albinism, which is due to a lack of tyrosinase activity.

Page 177:
FreeText
C. Page 334/364.
This patient is presenting with symptoms resembling Addison disease.
The patient's adrenal glands must not be responding to the ACTH so he is making more
ACTH (skin hyperpigmentation from MSH).
This is not a pituitary adenoma because this is not Cushing, where the patient would be
overweight (cushingoid habitus).

This patient has primary adrenal insufficiency and would present with hyponatremic volume
contraction and metabolic acidosis.
FreeText
In summary-
No Aldosterone = Lose Na+, Keep K+ & H+.
Keeping H+ means losing HCO3-.
Losing HCO3- means keeping Cl- via
basolateral HCO3/Cl exchanger found in PCT.
Metabolic acidosis.
Low Na and HCO3. High K, Cl, and H.
Think - Lose NaHCO3, Keep, HKCl.
No Cortisol = Anorexia, hypoglycemia, low BP.

Page 178:
Stamp
Palmar x-ray of right hand.
FreeText
To the left we have an X-ray of the right hand's palmar view.
A]Capitate B]Trapezoid C]Trapezium D]Scaphoid E]Lunate F]Pisiform G]Hamate

The hook of hamate injury can lead to ulnar nerve entrapment causing a distal ulnar nerve
injury presenting with paresthesias, numbness, and pain over the medial 2 fingers and claw
hand.
FreeText
C. Page 439/485.
You can tell that in the x-ray given, the middle bone, labelled E in our good xray, is
missplaced.
For once, NBME did an amazing job showing a pathological xray clearly missing
something. Always sing the song.
So Long To Pink, Here Comes The Thumb. Ive labelled them. The lateral view shows the
lunate poking out.
I like to remember that 2 bones lay on the radius. Scaphoid (most common fracture) and
lunate (causes acute carpal tunnel).
FreeText
s
FreeText
L??
FreeText
T,P
FreeText
H
FreeText
C
FreeText
T
FreeText
T
FreeText
L

Page 179:
FreeText
C. Page 598/673.
HOX genes organize the embryo in a craniocaudal direction by coding for transcription
factors.
Shh works by signal transduction.

Page 180:
FreeText
D. Page 151/153.
This patient is immunocompromised and has severe histoplasma induced pneumonia.
All systemic mycoses can form granuloma like TB so D makes sense.
Mycoses are not fought with neutrophils (for bacteria) or immune complexes (also for
bacteria).

Page 181:
FreeText
C. Page 136/137.
This patient is Strep pneumo induced bacterial meningitis,
S pneumo vaccination is extensively discussed in my microbiology video.
Children get the conjugate PCV13 is given to kids under age 5 years.
It is attached to Diptheria protein to elicit memory B cell development as well as T cell and
IgG, so it is immunogenic.
The PPSV23 is given mainly to adults and forms IgM because it is not immunogenic.

Page 182:
FreeText
D. Not in FA.
Again. Knowing NBME as soon as I saw that answer choice I was like this must be the
answer.
On top of that, nothing made me really want to pick B or E.
A was out of the question, he has no infection.
C was a possibility but they said most of his neurologic exam is perfect.

This video discusses phantom limb pain in detail --> https://youtu.be/5BlsoyE1CIw

It is apparently an extremely common sequela of amputation and mainly due to the


psychological stress your brain goes through
upon visualizing that the limb is physically missing.

Page 183:
FreeText
C. Page 107 of FA2019.
This patient has paroxysmal nocturnal hemoglobinuria. Its a nocturnal thing, so she has
dark urine "in the mornings."
G6PD deficiency is irrelevant because shes not sick, hasnt had fava beans, and is not in
any stress.
Abnormal cell morphology (such as hereditary spherocytosis) would have presented earlier
in her life, not acutely like this.
The "acidified serum test" is an obsolete test that could diagnose PNH by putting RBCs in
mild acid to see if the RBC are fragile.
It is not sensitive or specific so it sucks and isnt really done (get up to date on things NBME
wtf) and can even be positive in aplastic anemia.
Underproduction of globins would also have had presented earlier in her life.

Page 184:
FreeText
B. Page 74. Highlighted parts in image to right.
The most important fact they told was that administration of fructose or glycerol was not
increasing serum glucose.
The child has hypoglycemia and is making ketones.
When you give someone glycerol, its helpful in making glycerol-3-phosphate.
When you give someone fructose, that can also become glycerol-3-phosphate.

This G3P then becomes pyruvate which should be able to help go become acetyl CoA,
and become HMG CoA for ketogenesis. This is fine in the baby boy.

The same G3P can also go backwards for gluconeogenesis to make glucose.
For this baby it seems this path is not working.

Giving galactose helps because it skips this step regarding g3p.


Galactose becomes glucose via glycogenolysis and forms free glucose.
Stamp
Image
Highlight
Text Highlight
Highlight
Text Highlight
Highlight
Text Highlight

Page 185:
FreeText
B. Ethics and Treponema. Page 147/149.
This patient definitely has syphilis. The screening was done with RPR and the confirmatory
test was the MHA-TP.
That being said, the patient also has dementia but her memory impairment is "mild" and
her MMSE is not below 20/30.

Thus, it is best to first discuss HER health with HER first, in privacy.
If she fails to comprehend or if she herself says to discuss with her daughter present, then
you proceed from there.

Page 186:
FreeText
B. Page 640/722.
This man had his pituitary removed.
Today he has low testosterone, leading to azoospermia.
To raise his testosterone, we need him to make LH.
So inject LH.

Clomiphene or GnRH injections would fail because those rely on the pituitary.
Stamp
Image

Page 187:
FreeText
D. Page 246/256.
This patient had pufferfish with tetrodotoxin.
This toxin binds Na channels in nerve tissue, preventing depolarization.

Page 188:
FreeText
A. Page 256/266.

Having done clinical trials and being a clinical research coordinator at montefiore hospital
really made things clear first-hand.

This is a case series study. It is a type of medical research that uses demographics like
age, gender, and ethnicity in conjunction with information
on their diagnosis, treatment, and response to treatment over a period of time.

Some people thought this is C.

Physicians report new side-effects of medications in phase 4 of clinical trials, which is when
the drug is under post-marketing surveillance.
Patients take the drug because it is approved but if any unprecedented side effects show
up, physicians must report it to the pharma company
and submit the adverse event report to the IRB so all other places where this medication is
being administered can know of this. However, in this
scenario there would also be a control group. Drugs in phase 4 are offered often to clinical
trial patients where the patient consents that they
might be put in Arm A (treatment) or Arm B (placebo) and then side effects can pop up.

The NBME 20 question-


An investigator is conducting a study to identify potential risk factors for post-transplant
hypertension. The investigator selects post-transplant patients with
hypertension and gathers detailed information regarding their age, gender, preoperative
blood pressure readings, and current medications. The results of the
study reveal that some of the patients had been treated with cyclosporine. This study is
best described as which of the following? Case Series.
This study, which was compiled by aggregating several similar patient cases (i.e., all
patients with post-transplant hypertension), represents a case (or clinical)
series. A case series has no control group and samples individuals based on either
exposure (e.g., all patients treated with cyclosporine) or outcome (as seen
here). Without a control group, no measures of association between exposure and
outcome (e.g., relative risk, odds ratio) can be calculated.
Therefore, case series do not provide evidence of cause and effect.

Page 189:
FreeText
C.
This patient simply has a blister.
This has nothing to do with macrophages or necrosis, so A and D are out of the question.
This patient would not have eosinophelia in the area but the use of "perivascular" is a
distracter to make you think of choosing this answer.

This burn is simply going to cause an inflammatory response (think of it as the calor) and
that leads to post-venule capillary dilation.
Post venule is related to choice C.

Page 190:
FreeText
C. Page 669/769.
They confirmed that this is a malignant neoplasm, so you can cancel out D since that is
usually a coincidental, benign finding.
No risk factor or pathological image is given to make us think of malignant mesothelioma
(psammoma bodies, pleural thickening, asbestos).

Small cell carcinoma is central and carcinoid tumor is central with an excellent prognosis
(no mass effects of flushing, diarrhea, wheezing).

This is clearly a metastatic carcinoma because there are multiple (three) nodules found.
In the lung, metastases are more common than a primary neoplasm. It could be that this
lady has breast, colon, or bladder cancer.

Page 191:
FreeText
D. Page 274/262.

Great question to test the basic understanding of this biostatistical concept of confidence
interval.
When the 95% CI for an odds ratio or relative risk (this question) has 1 in it, the null
hypothesis is not rejected and the data is not significant.
Since the data is not significant, the p value must be more than 0.05, which is choice D.

The p value cannot be more than 1 because that would mean there is more than 100%
chance the null hypothesis is true, and that is unreal.
When data is significant, there is less than 5% of a chance that the data is by chance and
the null hypothesis is true.

Page 192:
FreeText
D.

Great question to test the understanding of partial vs full agonists. This concept plays a
bigger role when it comes to opioid receptor agonists.

Partial agonists have weak agonist activity on their own (thus in this case it causes HR to
increase, b-adrenergic effect) but when an actual
agonist is present (aka when you are exercising, you are producing NE and E that have full
b-agonist effects), partial agonist actually have
a mild antagonist effect (thus the heart rate decreases).

Page 193:
Stamp
Image
Stamp
Image
Stamp
Image
FreeText
D. Not in FA.
Lamins (not to be confused with laminins, which work with fibronectin and integrin to
connect cells to the basement membrane)
are a type 5 intermediate filament that interacts with the inner nuclear membrane protines
to form nuclear lamina on the inferior
nuclear envelope.
Really unnecessary, stupid question.

Page 194:
FreeText
A. Page 554/619.
This is a clear and good easy question. Manchausen syndrome by proxy.
The physician should report this parent because this is child abuse.
The high insulin with low C-peptide levels indicates that the girl was given exogenous
insulin.
This patient has ketosis and normal response to dextrose. Her metabolic pathways are
working fine.

Page 195:
FreeText
A. Page 349/383.
Great question about levothyroxine (T4) vs liothyronine (T3).
Excess levothyroxine intake will increase T4, which will be converted into T3 (this
conversion is not regulated).
Since there is so much active thyroid hormone, TSH will be extremely low and the thyroid
gland will not take up iodine because more
thyroid hormone production will not be necessary.

Some people may have felt that the answer would be B because ingesting T4 would
prevent the activation into T3 by peripheral 5'-deiodinase.
However, that enzyme just converts any T4 in circulation into T3, even if there is abundant
T3.
This is why taking excess levothyroxine can be a form of abusing the medication for weight
loss.
If 5'-deiodinase worked on a feedback mechanism, that would be amazing and overdosing
levothyroxine would have no major effect.

Liothyronine is T3, so when you ingest a lot of T3 you stop making more T4 thus you also
stop taking up iodine (choice E).

Page 196:
FreeText
A. Page 72. Page 232/242.
The question is on quite a hilarious method, but the concept is clear. If you want to increase
Km, what do you have to do?
Well, Km is inversely related to the affinity of the enzyme for its substrate. Raising Km
means that the enzyme is not attaching to the substrate.
This occurs when another substrate is more attractive, so the enzyme now attaches to that.

Remember the mnemonics from my book and videos and remember the lessons.

Km Increase = Kompetitive Inhibitors.

Think of Km as Kim kardashian and relationship-drama.


To increase the kim kardashian level of the scenario, introduce new competition of hot
women and the enzyme (the man, the one running everything)
will now be more attracted to the new hot woman. Etc.
Very easy good question :)
The hospital mustve ran out of fomepizole lol.

Page 197:
FreeText
D. Page 256/266.
Never get these wrong.
Phase 3 revolves around 10^3+ people (so 2000 or 5000, etc) who have the disease and
now you compare the new drug to the current standard.
This is why at this point it is double-blinded and randomized, because this patient MIGHT
get the current gold standard.
Whatever drug she gets, her data will be used to compare with other patients taking the
other drug and phase 3 checks for improvement.

Phase 0 trials are also known as human micro-dosing studies and are designed to speed
up the development of promising drugs or imaging agents
by establishing very early on whether the drug or agent behaves in human subjects as was
expected from pre-clinical studies.

Page 198:
FreeText
B. Page 553/618 of FA2019.
This patient is exhibiting the exact actions Sheldon Cooper of Big Bang Theory exhibited in
the show after he got married.
This is an avoidant personality where the person WANTS to have relationships with people
but is hypersensitive to criticism and
socially inhibited. This is in comparison to schizoid, where the person is like a robot and
does not care for relationships and is
perfectly fine on their own.

Antisocial is a trick answer that someone who hasnt mastered the personality disorders
would pick.
In normal, every day language, this patient could be described to be antisocial.
However, medically, this patient is not antisocial (a cluster B personality resonant of Cardi
B).
Antisocial people are very mean and have a disregard for the rights of others and can be
manipulative and hostile.

Page 199:
FreeText
C. Page 635/716.
This is the most viable answer choice to pick even without complete knowledge of all the
various breast cancers.
The patient has no mass on exam, no history of breast cancer, and doesnt take meds.
Bleeding from her nipple must mean intraductal problems and C is the best for that.
This is the most common cause of painless blood discharge from the nipple in a woman of
reproductive age.

A= Most common in premenopausal women. Would find painful lump possibly bilaterally.
Sclerosing adenosis has risk of cancer.
Epithelial hyperplasia has risk of cancer if you see atypical cells.
B= No mention in FA of a fibrosarcoma. But a fibroadenoma would be a well defined mobile
tender mass that grows in size with estrogen.
A fibrosarcoma is extremely rare and would also have a palpable mass.
D= Paget disease would have the eczematous patches over the areola.
E= Prolactinoma would simply cause milk discharge possibly but not blood discharge.

Page 200:
FreeText
G. Page 667/765.
This patient has a pneumothorax. The right lung is just gone and its all black due to air.
WHAT THE $#@! KIND OF CHEST XRAY IS THIS?!! WHY??
You cannot visualize the trachea at all but this is probably a tension pneumothorax with
tracheal deviation towards the left.
Honestly, none of the choices make sense once you realize this patient has a
pneumothorax, leaving only G to be even remotely feasible.
Arterial blood gas studies may show respiratory alkalosis caused by a decrease in CO2 as
a result of tachypnea
but later hypoxemia, hypercapnia, and acidosis would be seen.
Stamp
Image

Page 201:
Stamp
Normal lymphocyte on left, and reactive CD8+ cytotoxic T cell on right.

Can cause primary central nervous system lymphoma (PCNSL) in a patient with HIV and
multiple ring-enhancing lesions. Toxoplasma gondii is the #1 cause of ring enhancing
lesions, and PCNSL is the second most common cause of ring enhancing lesions.

PCNSL is a diffuse, large-cell non-Hodgkin lyphoma of B-cell origin that usually occurs as a
late complication of HIV infection. EBV is identified in almost all cases and tissue biopsy of
the brain would show abnormal B lymphocytes because T cell infiltration is not commonly
seen on brain tissue bopsy of patients with HIV who have PCNSL.
FreeText
C. Page 165/170 of FA2019.
I LOVE THIS QUESTION. Ive spoken about this topic numerous times and this is a great
question.
This patient has infectious mononucleosis due to HHV4 Epstein Barr Virus (remember that
Barr is 4 letters so HHV4).
The sheep agglutination is the heterophile antibody test.
These patients have reactive T cells presenting as atypical lymphocytes, while the infected
B cells look totally normal.
For viral infections, cytotoxic CD8+ T cells would be the ones reacting.

In comparison, AIDS patients with PCNSL have atypical lymphocytes that are actually the
infected B cells.

You might also like